Difference between revisions of "2023 AMC 10B Problems/Problem 18"

Line 12: Line 12:
 
III. gcd(𝑐, 210) = 1 if and only if gcd(𝑎, 14) = gcd(𝑏, 15) = 1.
 
III. gcd(𝑐, 210) = 1 if and only if gcd(𝑎, 14) = gcd(𝑏, 15) = 1.
  
== Solution (Guess and check + Contrapositive)==
+
== Solution 1 (Guess and check + Contrapositive)==
 
<math>I.</math>  Try <math>a=3,b=5 => c = 17\cdot15</math> which makes <math>\textbf{I}</math> false.
 
<math>I.</math>  Try <math>a=3,b=5 => c = 17\cdot15</math> which makes <math>\textbf{I}</math> false.
 
At this point, we can rule out answer A,B,C.
 
At this point, we can rule out answer A,B,C.
Line 25: Line 25:
 
~Technodoggo
 
~Technodoggo
  
==Solution==
+
==Solution 2==
  
 
The equation given in the problem can be written as
 
The equation given in the problem can be written as
Line 34: Line 34:
 
</cmath>
 
</cmath>
  
\textbf{First, we prove that Statement I is not correct.}
+
<math>\textbf{First, we prove that Statement I is not correct.}</math>
  
 
A counter example is <math>a = 1</math> and <math>b = 3</math>.
 
A counter example is <math>a = 1</math> and <math>b = 3</math>.
 
Thus, <math>{\rm gcd} (c, 210) = 3 \neq 1</math>.
 
Thus, <math>{\rm gcd} (c, 210) = 3 \neq 1</math>.
  
\textbf{Second, we prove that Statement III is correct.}
+
<math>\textbf{Second, we prove that Statement III is correct.}</math>
  
 
First, we prove the ``if'' part.  
 
First, we prove the ``if'' part.  
Line 62: Line 62:
 
Analogously, we can prove that <math>{\rm gcd}(b , 15) > 1</math>.
 
Analogously, we can prove that <math>{\rm gcd}(b , 15) > 1</math>.
  
\textbf{Third, we prove that Statement II is correct.}
+
<math>\textbf{Third, we prove that Statement II is correct.}</math>
  
 
This is simply a special case of the ``only if'' part of Statement III. So we omit the proof.
 
This is simply a special case of the ``only if'' part of Statement III. So we omit the proof.
  
 
All analysis above imply
 
All analysis above imply
\boxed{\textbf{(E) II and III only}}.
+
<math>\boxed{\textbf{(E) II and III only}}.</math>
  
 
~Steven Chen (Professor Chen Education Palace, www.professorchenedu.com)
 
~Steven Chen (Professor Chen Education Palace, www.professorchenedu.com)

Revision as of 18:24, 15 November 2023

Problem

Suppose 𝑎, 𝑏, and 𝑐 are positive integers such that $\dfrac{a}{14}+\dfrac{b}{15}=\dfrac{c}{210}$.

Which of the following statements are necessarily true?

I. If gcd(𝑎, 14) = 1 or gcd(𝑏, 15) = 1 or both, then gcd(𝑐, 210) = 1.

II. If gcd(𝑐, 210) = 1, then gcd(𝑎, 14) = 1 or gcd(𝑏, 15) = 1 or both.

III. gcd(𝑐, 210) = 1 if and only if gcd(𝑎, 14) = gcd(𝑏, 15) = 1.

Solution 1 (Guess and check + Contrapositive)

$I.$ Try $a=3,b=5 => c = 17\cdot15$ which makes $\textbf{I}$ false. At this point, we can rule out answer A,B,C.

$II.$ A => B or C. equiv. ~B AND ~C => ~A. Let a = 14, b=15 (statisfying ~B and ~C). => C = 2*210. which is ~A.

$II$ is true.

So the answer is E. $\boxed{\textbf{(E) } II \text{ and } III \text{only}.}$ ~Technodoggo

Solution 2

The equation given in the problem can be written as \[ 15 a + 14 b = c. \hspace{1cm} (1) \]

$\textbf{First, we prove that Statement I is not correct.}$

A counter example is $a = 1$ and $b = 3$. Thus, ${\rm gcd} (c, 210) = 3 \neq 1$.

$\textbf{Second, we prove that Statement III is correct.}$

First, we prove the ``if part.

Suppose ${\rm gcd}(a , 14) = 1$ and ${\rm gcd}(b, 15) = 1$. However, ${\rm gcd} (c, 210) \neq 1$.

Thus, $c$ must be divisible by at least one factor of 210. W.L.O.G, we assume $c$ is divisible by 2.

Modulo 2 on Equation (1), we get that $2 | a$. This is a contradiction with the condition that ${\rm gcd}(a , 14) = 1$. Therefore, the ``if part in Statement III is correct.

Second, we prove the ``only if part.

Suppose ${\rm gcd} (c, 210) \neq 1$. Because $210 = 14 \cdot 15$, there must be one factor of 14 or 15 that divides $c$. W.L.O.G, we assume there is a factor $q > 1$ of 14 that divides $c$. Because ${\rm gcd} (14, 15) = 1$, we have ${\rm gcd} (q, 15) = 1$. Modulo $q$ on Equation (1), we have $q | a$.

Because $q | 14$, we have ${\rm gcd}(a , 14) \geq q > 1$.

Analogously, we can prove that ${\rm gcd}(b , 15) > 1$.

$\textbf{Third, we prove that Statement II is correct.}$

This is simply a special case of the ``only if part of Statement III. So we omit the proof.

All analysis above imply $\boxed{\textbf{(E) II and III only}}.$

~Steven Chen (Professor Chen Education Palace, www.professorchenedu.com)